Subtract the polynomials
(3x² + 2x − 5) – (4x² – 3x − 4)


A:-7x²+5x-9

B:7x²-x-9

C:x²-x-9

D:-x²+5x-1

please helpppp asap​

Answers

Answer 1

Answer:

-x² + 5x - 1

Step-by-step explanation:

separate the x², the x, and the constants (numbers without any x).

3x² - 4x² = -x²

2x - -3x = 2x + 3x = 5x

-5 - -4 = -5 + 4 = -1.

we have -x² + 5x - 1


Related Questions

Need help with first four

Answers

Answer:

For 1 is it 1 hour and 15 mins?

Step-by-step explanation:

6 by 32 = 30 mins

12 by 32 = 1 hr

13 by 32 = 15

1hr + 15 mins = 1hr 15 mins

Sorry if incorrect

which construction is being demonstrated? your answer: constructing a perpendicular bisector. constructing a line perpendicular to a line through a point not on the line. constructing a line parallel to a line through a point not on the line. constructing a line perpendicular to a line through a point on the line.

Answers

The construction being demonstrated is constructing a perpendicular bisector.

In this construction, a line is drawn to bisect a given line segment and is perpendicular to that line segment. The perpendicular bisector divides the line segment into two equal parts and creates a right angle at the point of intersection. It is useful in various geometric constructions and applications, such as finding the midpoint of a line segment or constructing equilateral triangles. By constructing a perpendicular bisector, we ensure that the distances from any point on the line to the endpoints of the line segment are equal.

Know more about perpendicular bisector here:

https://brainly.com/question/29132624

#SPJ11

x^4-5x^3+7x^2-5x+6=0

Answers

Answer:Therefore, the solutions to the equation x^4 - 5x^3 + 7x^2 - 5x + 6 = 0 are x = 0, x = 1, x = 5.

Step-by-step explanation:

To solve the equation x^4 - 5x^3 + 7x^2 - 5x + 6 = 0, we can use factoring by grouping.

First, we can group the first two and last two terms:

x^4 - 5x^3 + 7x^2 - 5x + 6 = (x^4 - 5x^3) + (7x^2 - 5x + 6)

Next, we can factor out x^3 from the first group and factor out 1 from the second group:

(x^3(x - 5)) + (7x^2 - 5x + 6)

Now, we can group the last two terms of the second group:

x^3(x - 5) + (7x^2 - 3x - 2x + 6)

Then, we can factor out 1 from the terms inside the parentheses and group them:

x^3(x - 5) + (7x^2 - 3x) + (-2x + 6)

Now, we can factor out x from the second and third groups:

x^3(x - 5) + x(7x - 3) - 2( x - 3)

We can simplify the third group by distributing the negative sign:

x^3(x - 5) + x(7x - 3) - 2x + 6

Finally, we can combine the second and third groups:

x^3(x - 5) + x(7x - 5) + 6

So, the factored form of the equation x^4 - 5x^3 + 7x^2 - 5x + 6 = 0 is:

(x^3(x - 5) + x(7x - 5) + 6) = 0

This equation can be solved by setting each factor equal to zero and solving for x:

x^3(x - 5) + x(7x - 5) + 6 = 0

(x^3 - 7x^2 + 5x) + (6 - 5x) = 0

x(x^2 - 7x + 5) - (5x - 6) = 0

x(x - 5)(x - 1) - (5x - 6) = 0

x(x - 5)(x - 1) = 5x - 6

x^3 - 6x^2 + 10x - 6 = 5x - 6

x^3 - 6x^2 + 5x = 0

x(x^2 - 6x + 5) = 0

x(x - 1)(x - 5) = 0

Therefore, the solutions to the equation x^4 - 5x^3 + 7x^2 - 5x + 6 = 0 are x = 0, x = 1, x = 5.

Use the formula to find the surface area of the figure. Show your work.

Answers

230 inches to the power of 2

TANK A large tank is currently holding 12,000 gallons of water. The water will drain at a constant rate of 100 gallons per minute until the volume of water in the tank is 4,000 gallons. Write an equation to find the number of minutes m it will take for the specified amount of water to drain.

Answers

The equation to find the number of minutes m is 12000 - 100m = 4000

Writing an equation to find the number of minutes m

From the question, we have the following parameters that can be used in our computation:

Initial volume  = 12000 gallons

Rate of draining = 100 gallons per minute

Final volume of water = 4000 gallons

The equation to find the number of minutes m is represented as

f(m) = Initial volume - Rate of draining * m

So, we have

f(m) = 12000 - 100m

When the volume is at 4000 gallons, we have

12000 - 100m = 4000

Hence, the equation to find the number of minutes m is 12000 - 100m = 4000

Read more about linear functions at

https://brainly.com/question/15602982

#SPJ1

I need to know everything

Answers

The values of x, y and z in the parallelogram 1 are x = 80, y = 100 and z = 80

Parallelogram 2: x = 130, y = 130 and z = 130 Parallelogram 3: x = 90, y = 60 and z = 60 Parallelogram 4: x = 100, y = 80 and z = 80Parallelogram 5: x = 28 , y = 112 and z = 28

Finding the values of x, y and z in the parallelograms

Parallelogram 1

Adjacent angles of a parallelogram add up to 180

So, we have

x = 180 - 100

x = 80

Opposite angles are equal

So, we have

y = 100

z = 80

Using the above theorem, we have the values of x, y and z in the other parallelograms to be

Parallelogram 2

x = 180 - 50

x = 130

y = 130

z = 130 --- by corresponding angle theorem

Parallelogram 3

x = 90 --- by vertical angle theorem

Then, we have

y = 60 --- sum of angles in a triangle

z = 60 --- by corresponding angle theorem

Parallelogram 4

x = 100

y = 80

z = 80 --- by corresponding angle theorem

Parallelogram 5

y = 112

x = 180 - 112 - 40 --- sum of angles in a triangle

x = 28

z = 28  --- by corresponding angle theorem

Read more about angles at

https://brainly.com/question/25716982

#SPJ1

Determine which of the following statements is true for the function g(z) = 4x³ - 3r + 2r³ - 1.
A

D
As x→ ∞o, f(x) → ∞o and as x→→∞, f(x)→-00.
As x → ∞o, f(x) → ∞o and as x→-∞, f(x)→ 00.
As x→ ∞o, f(x)--∞o and as x→-∞, f(x) →∞0.
As x→ ∞, f(x)-- and as x→-∞, f(x)→-00.

Answers

The correct statement is:
As x → ∞, f(x) → ∞ and as x → -∞, f(x) → -∞.

I understand that you are asking about the behavior of the function g(z) as x approaches positive and negative infinity. Let's analyze the given function:
g(z) = 4x³ - 3r + 2r³ - 1
First, I believe "r" should be "x" to keep the variables consistent. So, the corrected function is:
g(x) = 4x³ - 3x + 2x³ - 1
Now, let's find the end behavior of this function as x approaches positive and negative infinity.
As x → ∞:
The highest degree term, x³, will dominate the function's behavior. The coefficients for x³ are 4 and 2, so the function will behave like 6x³. Since this term is positive and has an odd exponent, as x → ∞, f(x) → ∞.
As x → -∞:
Again, the x³ term will dominate the function's behavior. Since the exponent is odd, as x → -∞, f(x) → -∞.
So, the correct statement is:
As x → ∞, f(x) → ∞ and as x → -∞, f(x) → -∞.

For more questions on correct statement

https://brainly.com/question/31250703

#SPJ11

The radius of a basketball is 9 inches. What is the volume of the basketball? Round to the nearest tenth.

Answers

By definition of volume of sphere, The volume of the basketball is,

V = 3052.08 inches³

We have to given that;

The radius of a basketball is 9 inches.

Since, We know that;

To multiply means to add a number to itself a particular number of times. Multiplication can be viewed as a process of repeated addition.

And,  We know that;

Volume of sphere = 4/3πr³

Where, r is radius of sphere.

And, pi is stand for 3.14.

Hence, We get;

The volume of the basketball is,

⇒ V = = 4/3πr³

Substitute radius (r) = 9 inches, pi = 3.14 in above equation, we get;

⇒ V = 4/3 × 3.14 × 9³ inches³

⇒ V = 4/3 × 3.14 × 243 inches³

⇒ V = 3052.08 inches³

Therefore, The volume of the basketball is,

⇒ V = 3052.08 inches³

Learn more about the circle visit:

https://brainly.com/question/24810873

#SPJ1

<
←+
-6
6
4
U ←
2
-4
-6-
2
6
→x
Write an equation that represents the line.
Use exact numbers.

Answers

To find the equation in slope-intercept form, y = mx + b, we need to first find the slope using
    [tex]m=\dfrac{y_2-y_1}{x_2-x_1}[/tex]

 

In this case we have the two points (1,2) and (4,4), so our calculation looks like:

    [tex]m=\dfrac{4-2}{4-1} = \dfrac{2}{3}[/tex]
Notice this is the same "rise over run" that we can count out on the graph.  If we start at (1,2) and go "up 2 units, right 3 units", then we end up at the point (4,4).

 

Now we need to find b.  We have [tex]y = \frac{2}{3}x+b[/tex], so we need to substitute in one of the points to find b.  Let's use (1,2):

           [tex]2 = \frac{2}{3}\cdot 1+b[/tex]

           [tex]2 = \frac{2}{3}+b[/tex]

    [tex]2 - \frac{2}{3} =b[/tex]

           [tex]\frac{4}{3} =b[/tex]

 

This gives us our equation: [tex]y = \frac{2}{3}x+\frac{4}{3}[/tex]

A person who owns a 2/3 of a land sells 1/4 of it express the land sold as a fraction of the whole land

Answers

If a person owns 2/3 of a land and sells 1/4 of it, the fraction of the whole land sold can be expressed as 1/4 divided by 2/3.

To divide fractions, we invert the second fraction and multiply. Therefore, we have:

1/4 ÷ 2/3 = 1/4 × 3/2 = 3/8

So, the fraction of the whole land sold is 3/8. This means that the person has 2/3 - 3/8 = 5/24 of the original land remaining after selling 1/4 of it.

To see why this is the case, we can visualize the original land as a pie chart. If the person owns 2/3 of the land, then 1/3 of the land belongs to someone else. If the person sells 1/4 of their share, they are effectively selling 1/4 of 2/3 of the land, or 2/12 of the land. This represents a portion of the pie chart that is 2/12 of the total pie, or 1/6. Therefore, the person now owns 5/6 of the pie chart, or 5/24 of the original land.

To learn more about fractions click here, brainly.com/question/10354322

#SPJ11

plot points H I and J on the coordinate plane​

Answers

In order to plot points H I and J on the coordinate planeIdentify the x-coordinate and y-coordinate of each point (H, I, and J). For example, let's say the coordinates are as follows:

Point H: (xH, yH)

Point I: (xI, yI)

Point J: (xJ, yJ)

How to explain the information

Locate the origin (0, 0) on the coordinate plane. This is the starting point for plotting any point.

Move along the x-axis according to the x-coordinate of each point. If the x-coordinate is positive, move to the right; if it's negative, move to the left. Mark a point on the x-axis at the appropriate location.

Move along the y-axis according to the y-coordinate of each point. If the y-coordinate is positive, move upward; if it's negative, move downward. Mark a point on the y-axis at the appropriate location.

The point where the x and y axes intersect is the location of the point. Mark that point on the coordinate plane.

Learn more about coordinate on

https://brainly.com/question/29765572

#SPJ1

How to Plot points H I and J on the coordinate plane

who can determine the perimeter of the following regular nonagon??

Answers

The perimeter of the regular nonagon is approximately 29.7 feet.

The apothem of a regular nonagon divides each of its interior angles into two congruent angles.

Therefore, each of the interior angles of the nonagon measures:

(180 - 360/9)/2 = 140 degrees

The sum of the interior angles of a nonagon is (9-2) * 180 = 1260 degrees. Therefore, the measure of each exterior angle of the nonagon is:

360/9 = 40 degrees

In a regular nonagon, all the sides and angles are congruent, so we can divide it into 9 congruent isosceles triangles. Each of these triangles has base s and height a, and its legs are given by:

l =√s² - (a/2)²

The perimeter P of the nonagon is given by:

P = 9s

Substituting the values of s and a, we get:

l = √3.3²- (1.65/2)²

= 3.018 ft (rounded to 3 decimal places)

P = 9(3.3) = 29.7 ft

Therefore, the perimeter of the regular nonagon is approximately 29.7 feet.

To learn more on Area click:

https://brainly.com/question/20693059

#SPJ1

A radio station runs a promotion at an auto show with a money box with 13 ​$50 ​tickets, 11​$25 ​tickets, and ​11$5 tickets. The box contains an additional ​20 "dummy" tickets with no value. Three tickets are randomly drawn. Find the probability that exactly two ​$50 prizes and no other money winners are chosen

Answers

The probability of choosing exactly two $50 tickets and no other money winners is:

34,320 / 21,455 ≈ 0.

we can use the hypergeometric    probability   distribution to solve this problem, since we are drawing without replacement from a finite population of tickets with different values.

the total number of tickets in the box is:

13 + 11 + 11 + 20 = 55

the number of ways to choose exactly two $50 tickets from the 13 available is:

${13 \choose 2} = 78$

the number of ways to choose one dummy ticket from the 20 available is:

${20 \choose 1} = 20$

the number of ways to choose one ticket from the remaining non-$50 tickets is:

11 + 11 = 22

, the total number of ways to choose exactly two $50 tickets and no other money winners is:

78 x 20 x 22 = 34,320

the number of ways to choose any three tickets from the 55 available is:

${55 \choose 3} = 21,455$ 60 or about 60%

Learn more about probability here:

https://brainly.com/question/32117953

#SPJ11

After heating up in a teapot, a cup of hot water is poured at a temperature of
20
3

203

F. The cup sits to cool in a room at a temperature of
6
9

69

F. Newton's Law of Cooling explains that the temperature of the cup of water will decrease proportionally to the difference between the temperature of the water and the temperature of the room, as given by the formula below:

=


+
(

0



)




T=T
a

+(T
0

−T
a

)e
−kt



=
T
a

= the temperature surrounding the object

0
=
T
0

= the initial temperature of the object

=
t= the time in minutes

=
T= the temperature of the object after

t minutes

=
k= decay constant

The cup of water reaches the temperature of
18
5

185

F after 1.5 minutes. Using this information, find the value of

k, to the nearest thousandth. Use the resulting equation to determine the Fahrenheit temperature of the cup of water, to the nearest degree, after 4.5 minutes.

Enter only the final temperature into the input box.

Answers

The temperature of the water after 4.5 minutes is approximately 153°F.

How to find the Fahrenheit temperature of the cup of water, to the nearest degree, after 4.5 minutes.

Using Newton's Law of Cooling to find the value of the decay constant k: T = [tex]Ta + (T0 - Ta) * e^-k*t[/tex]

Substituting the given values, we get:

185 = [tex]69 + (203 - 69) * e^-k*1.5[/tex]

Simplifying, we get:

[tex]116 = 134 * e^ \\^{-1.5k}[/tex]

Dividing both sides by 134, we get:

[tex]0.8657 = e^{-1.5k}[/tex]

Taking the natural logarithm of both sides, we get:

ln(0.8657) = -1.5k

Solving for k, we get:

k ≈ 0.232

Therefore, the value of the decay constant is approximately 0.232.

To find the temperature of the water after 4.5 minutes, we can use Newton's Law of Cooling again, with t = 4.5:

[tex]T = Ta + (T0 - Ta) * e^-k*t[/tex]

[tex]T = 69 + (203 - 69) * e^-0.232*4.5[/tex]

T ≈ 153°F

Therefore, the temperature of the water after 4.5 minutes is approximately 153°F.

Learn more about decay constant at https://brainly.com/question/31314266

#SPJ1

smith is in jail and has 3 dollars; he can get out on bail if he has 8 dollars. a guard agrees to make a series of bets with him. if smith bets a dollars, he wins a dollars with probability .4 and loses a dollars with probability .6. find the probability that he wins 8 dollars before losing all of his money if

Answers

Therefore, the probability that Smith wins 8 dollars before losing all of his money is approximately 0.0479.

To solve this problem, we can use a probability tree diagram to visualize the different possible outcomes.

At each stage, Smith either wins a dollars or loses a dollars, until he either reaches 8 dollars (and wins) or 0 dollars (and loses). We can calculate the probability of each outcome by multiplying the probabilities of the branches leading to that outcome.

Starting with 3 dollars, there are two possible outcomes:

Smith wins a dollar with probability 0.4, leaving him with 4 dollars.

Smith loses a dollar with probability 0.6, leaving him with 2 dollars.

From 4 dollars, there are three possible outcomes:

Smith wins a dollar with probability 0.4, leaving him with 5 dollars.

Smith loses a dollar with probability 0.6, leaving him with 3 dollars.

Smith wins 4 dollars with probability 0.4 * 0.4 = 0.16, leaving him with 7 dollars.

From 5 dollars, there are two possible outcomes:

Smith wins a dollar with probability 0.4, leaving him with 6 dollars.

Smith wins 3 dollars with probability 0.4 * 0.4 = 0.16, leaving him with 8 dollars.

From 6 dollars, there are two possible outcomes:

Smith wins 2 dollars with probability 0.4 * 0.4 = 0.16, leaving him with 8 dollars.

Smith loses a dollar with probability 0.6, leaving him with 5 dollars.

From 7 dollars, there is one possible outcome:

Smith wins 1 dollar with probability 0.4, leaving him with 8 dollars.

Therefore, the probability that Smith wins 8 dollars before losing all of his money is the sum of the probabilities of the outcomes that lead to winning 8 dollars, which is:

0.4 * 0.6 * 0.4 * 0.4 + 0.4 * 0.6 * 0.4 * 0.6 * 0.4 + 0.4 * 0.6 * 0.4 * 0.4 * 0.6 * 0.16 + 0.4 * 0.6 * 0.4 * 0.4 * 0.4 * 0.4 * 0.16 = 0.047872

To know more about probability,

https://brainly.com/question/13690215

#SPJ11

Explain step by step​

Answers

Answer:

buying price = $11764.70

Step-by-step explanation:

selling price = $10000

loss = 15%

85% = 10000

100% = 10000/85 × 100

= $ 11764.70

Help will give BRAINLYST If a certain soil sample contains 200 grams of water on July 1st,which equat describes the relationship between y amount of water in grams,and t time in weeks after July 1st

Answers

The required equation is y = 200 - (0.025)t

Hence option C is correct.

According to the given information:

The soil's water content is dropping by 2.5% weekly.

And here, Begin with the 200 gram of water that were initially present in the soil sample on July 1.

Then deduct the weekly water loss,

Which is determined by multiplying the original water amount by 25%  and the number of weeks (t).

Now forming the equation,

⇒ y = 200 - (2.5/100)t

⇒ y = 200 - (0.025)t

Hence, the expression be,

y = 200 - (0.025)t

Learn more about the mathematical expression visit:

brainly.com/question/1859113

#SPJ1

On monday 5/8 inches of rain fell in 2/3 hour. What is the value of the excretion?

Answers

The value of the excretion is equal to 15 / 16 inches of precipitation per hour.

How to find the value of the excretion

In this question we find that a precipitation of 5 / 8 inches of rain is registered in a time of 2 / 3 hour and we are asked to find how many precipitation is registered in a time of an hour, that is, the value of the excretion. The excretion can be found by cross multiplication:

r = (5 / 8 in) / (2 / 3 h)

r = 15 / 16 in / h

A precipitation of 15 / 16 inches in a time of an hour is the value of the excretion.

To learn more on cross multiplication: https://brainly.com/question/27812904

#SPJ1

The figure shows △GHJ and △PQR on a coordinate plane.

a. Explain why the triangles are congruent using the ASA Triangle Congruence Theorem.

b. Explain why the triangles are congruent using rigid motions

Answers

Using the ASA Triangle Congruence Theorem, we can prove that triangles GHJ and PQR are congruent because they have Angle G and angle P are congruent, both being right angles.

The two triangles are congruent by rigid motions.

Using the ASA Triangle Congruence Theorem, we can prove that triangles GHJ and PQR are congruent because they have:

Angle G and angle P are congruent, both being right angles.

Side GH and side PQ are congruent, both having a length of 5 units.

Angle H and angle Q are congruent, both having a measure of 63.43 degrees (rounded to two decimal places).

Since the two triangles have two congruent angles and a congruent side between them, they are congruent by the ASA Triangle Congruence Theorem.

b. We can also prove that triangles GHJ and PQR are congruent using rigid motions.

Specifically, we can use a translation followed by a reflection to map triangle GHJ onto triangle PQR.

Translation: We can translate triangle GHJ 2 units to the left and 3 units down to get triangle G'H'J', where G'(-2, 1), H'(-2, -2), and J'(2, -2).

Reflection: We can reflect triangle G'H'J' across the x-axis to get triangle PQR, where P(-2, -4), Q(-2, -1), and R(2, -1).

To learn more on Graph click:

https://brainly.com/question/17267403

#SPJ1

Isabell run 7 miles in 80 minutes at the same rate how many miles would she run in 64 minutes

Answers

7(64)/80 = 5.6 miles{ANSWER}♥️

two people leave their shared apartment at the same time. during a 500 second interval, one jogs 10 blocks south at a constant rate, whereas the second walks 5 blocks west, again, at a constant rate. what is their speed relative to each other during this interval? assume each block corresponds to 100 meters.

Answers

The relative speed between the jogger and the walker during the 500 second interval is 2.24 m/s.

To solve this problem, we need to first find the distances traveled by each person.

The jogger travels 10 blocks x 100 meters/block = 1000 meters south.

The walker travels 5 blocks x 100 meters/block = 500 meters west.

Using the Pythagorean theorem, we can find the distance between them: √((1000m)² + (500m)²) = 1118.03 meters.

To find their relative speed, we divide this distance by the time interval: 1118.03 meters / 500 seconds = 2.24 m/s.

Therefore, their speed relative to each other during this interval is 2.24 m/s.

The jogger traveled 1000 meters south, while the walker traveled 500 meters west. Using the Pythagorean theorem, the distance between them is 1118.03 meters. Dividing this distance by the time interval of 500 seconds, we get their relative speed, which is 2.24 m/s.

: The relative speed between the jogger and the walker during the 500 second interval is 2.24 m/s.

To know more about Pythagorean theorem visit:

brainly.com/question/14930619

#SPJ11

Calculate the length of AC to 1 decimal place

Answers

The length of AC by the given data is about 11.0 cm.

We are given that;

ABCD is a trapezium AB=16cm, AD=11cm, BC=4cm

Now,

We can use these values to find DC. Since ABCD is a trapezium, we know that AB and DC are parallel. Therefore, the distance between them is constant. We can write:

AB−BC=AD−DC

Plugging in the given values, we get:

16−4=11−DC

Solving for DC, we get:

DC=11−12=−1

We can ignore the negative sign since we are only interested in the length of DC. So, DC = 1 cm.

Now we can plug in AD = 11 cm and DC = 1 cm into the Pythagorean theorem and get:

AC2=112+12

AC2=122

Taking the square root of both sides, we get:

AC=122​≈11.045

Rounding to one decimal place, we get:

AC≈11.0 cm

Therefore, by algebra the answer will be 11.0 cm.

More about the Algebra link is given below.

brainly.com/question/953809

#SPJ1

Simplify, (Write each expression without using the absolute value symbol)
x-(-12) if x<-12

Answers

Answer:

If x < -12, then (-x) > 12, and we have:

x - (-12) = x + 12

So, the simplified expression without using the absolute value symbol is:

x + 12

Step-by-step explanation:

Answer:

Since x is less than -12, then x-(-12) = x+12.

Here is a table of values for x and x-(-12):

x | x-(-12)

-13 | -1

-14 | -2

-15 | -3

...

Step-by-step explanation:

Using the results of the survey, out of 9,000 customers how many should the manager expect to visit late on weekends?

Answers

According to the survey, 40% of customers visit late on weekdays and 60% visit late on weekends. Therefore, out of 9,000 customers, we can expect 60% of them to visit late on weekends.

To find out the exact number, we can use the following calculation:

60% of 9,000 = (60/100) x 9,000 = 5,400

Therefore, the manager can expect around 5,400 customers to visit late on weekends out of a total of 9,000 customers.

Learn more about survey here: brainly.com/question/31772865

#SPJ11

Use the drop-down menus to choose steps in order to correctly solve
4k−6=−2k−16−2
for k
.

Answers

Answer:

-6

Step-by-step explanation:

4k-6=2k-16-2

-16-2= -18

4k-6=2k-18

+6= +6 from both sides

4k=2k-12

-2k = -2k from both sides

2k = -12

/2 /2

k= -6

given f(x)=3x+2 and g(x)= √x-1, determine the following: g(f(8))=

Answers

The function operation g(f(8) in the given functions f(x) = 3x+2 and g(x) = √(x-1) is 5.

What is the function operation g(f(8) in the given functions?

A function is simply a relationship that maps one input to one output.

Given that:

f(x) = 3x + 2g(x) = √( x - 1 )g(f(x)) = ?

First, set up the composite result function:

Evaluate g( 3x + 2 ) by substituting in the value of f into g.

g( 3x + 2 ) = √( ( 3x + 2 ) - 1 )

Simplify

g( 3x + 2 ) = √( 3x + 2 - 1 )

g( 3x + 2 ) = √( 3x + 1 )

Evaluate the result function by replacing the x with 8.

g( f(x) ) = √( 3(8) + 1 )

g( f(x) ) = √( 24 + 1 )

g( f(x) ) = √( 25 )

g( f(x) ) = 5

Therefore, the composite result function g( f(x) ) is 5.

Learn more about functions here: https://brainly.com/question/28780365

#SPJ1

PROBLEM SOLVING: Write the answer corresponds to the problem. REMINDERS

If the answer requires decimals, express your answer in 2 decimal places. If your answer is more than 3 digits, DO NOT INCLUDE THE COMMA. Compute for the mean expenses of the XYZ Corporation given the following:



Name Total Sales Total Expenses
Quezon City 14,950. 00 4,933. 50
Caloocan City 18,290. 00 6,035. 70
Marikina City 37,200. 00 12,276. 00
Cebu City 18,900. 00 6,237. 00
Davao City 45,000. 00 14,850. 00
Mandaluyong City 23,000. 00 7,590. 00
Cavite 22,000. 00 7,260. 00
Laguna 21,000. 00 6,930. 00
Manila 66,000. 00 21,780. 00
Iloilo 34,000. 00 11,220. 0

Answers

The mean expenses of the XYZ Corporation is 9,811.22.

To compute for the mean expenses of the XYZ Corporation, we need to add up all the total expenses of each city and divide it by the total number of cities.

Total Expenses = 4,933.50 + 6,035.70 + 12,276.00 + 6,237.00 + 14,850.00 + 7,590.00 + 7,260.00 + 6,930.00 + 21,780.00 + 11,220.00
Total Expenses = 98,112.20

Number of Cities = 10

Mean Expenses = Total Expenses / Number of Cities
Mean Expenses = 98,112.20 / 10
Mean Expenses = 9,811.22

The mean expenses of the XYZ Corporation is 9,811.22. Since the answer does not require decimals, we do not need to express it in two decimal places. However, we need to follow the reminder that if the answer is more than three digits, we should not include the comma.

Learn more about mean here:

https://brainly.com/question/31101410

#SPJ11

425 divided by 6 so i get the answer of 7.833333333 and so on but that answer is wrong i don’t understand how

Answers

425 divide by 6 = 70.8333333…
But you round off and get 70.83 or 70.8

The solution of expression after divide is, 70.83333...

We have to give that,

Divide 425 by 6.

Now, Divide the numbers as,

425 ÷ 6

6 ) 425 ( 70.833

   - 42

  --------

        050

         - 48

       -----------

               20

             - 18

             --------

                 20

               - 18

             ----------

                    2

Hence, The solution of expression after divide is, 70.83333...

To learn more about the divide visit:

https://brainly.com/question/28119824

#SPJ6

A cylinder has a height of 18 inches and a diameter of 40 inches. What is its volume? Use ​ ≈ 3.14 and round your answer to the nearest hundredth.

Answers

Answer:

≈226.1947

Step-by-step explanation:

have a great day and thx for your inquiry :)

What is the answer to this question??

Answers

The missing length indicated has the value given as follows:

x = 25.

What is the geometric mean theorem?

The geometric mean theorem states that the length of the altitude drawn from the right angle of a triangle to its hypotenuse is equal to the geometric mean of the lengths of the segments formed on the hypotenuse.

The bases in this problem are given as follows:

x and 144.

The length of the altitude segment is given as follows:

60.

60 is the geometric mean of x and 144, hence the value of x is obtained as follows:

144x = 60²

x = 3600/144

x = 25.

More can be learned about the geometric mean theorem at brainly.com/question/10216660

#SPJ1

Other Questions
how do the landscape paintings of john constable evoke themes of romanticism in art? . Francois has 8 feet of trim. He ismaking 3 picture frames. Write afraction that represents the number offeet of trim that can be used for eachpicture frame. Fill in the bubble beforeeach number that is correct.Pls help its a grade 4.Look at the picture below.What does the diagram show?A. refractionB. absorptionC. reflectionD. diffraction (x - 2)/(x + 2) - (x + 2)/(x - 2) Which utilities can be used to manage printers? (Choose all that apply.)a. the Devices and Printers appletb. Computer Managementc. Device Managerd. Print Management snap-in With the above statements in mind, make a research project on the cost and revenue of a Bricklaying business in your area. The aim of a business is produce Bricks at lower cost per unit and to create more jobs to the local people around the area. 1 Write a short report (3) a cause of female infertility is failure of ovulation due to hyposecretion of hormones from the Which of the following is not an example of a mnemonic technique that emphasizes organization?a. Keyword methodb. Hierarchyc. First-letter techniqued. Narrative technique A golfer played 7 rounds of golf with the following scores: 1 round at 3 over par, 2 rounds with 1 over par, 3 rounds with 3 under par, and the last round at 1 under par. How did she finish with respect to par? The ability to choose a pricing policy and apply it to your product or service is an important skill for all entrepreneurs. This activity will provide practice doing basic pricing calculations, using your product costs and your markup.Step 1: Identify Pricing Risk FactorsDetermine risks associated with your pricing objectives. Look at how the price is affected by:supply and demandthe competitioneconomic conditionsgovernment regulationsHow does the target market itself affect your pricing?Open a document and title it Pricing.Create a heading on the document and label it Risk Factors.Write four to five sentences that describe your product in relation to each of the specific risk factors.Step 2: Determine Your Pricing Objective(s)Determine your pricing objective. Note that you can choose to maximize sales, profits, market share, or simply to cover your costs and earn a modest profit.Create a second heading on the document and label it Pricing Objectives.Write your pricing objective in one or two sentences. Note that you may decide to optimize on more than one objective at the same time. Tommy and his friends are planning a trip to the Mega Vault trampoline park. The employeeTommy spoke to on the phone said the total cost for all 5 of them would be $85. This includesa $3 fee added to each entrance ticket to cover the cost of a pair of grip socks.Which equation can you use to find c, the cost of an entrance ticket?5(c + 3) = 855c + 3 - 853c+5 853(c+5)= 85 An injection of ACTH would cause an increase in the release of which of the following hormones?A) thyroxinB) glucocorticoidsC) growth hormoneD) antidiuretic hormone a single allele that controls more than one character is said to be __________. the basin in the floor of the cranium that accommodates the temporal lobe of the brain is the ________graphy is a series of x-ray images made to show an organ in depth. In order to implement Comparable in a class, what method(s) must be defined in that class? A. both compares and equals B. both less than and greater Than C. compare To D. equals E. compares if you were interested in upper paleolithic cave paintings, you would probably travel to Select all that apply Which of the following would be found on a statement of stockholders' equity? (Select all that apply.) a Net Income b Additional Paid-in Capital c Stock Issuances d Dividends Payable e Treasury Stock f Dividends the social scientist who argued that religion brought a new vitality to people's lives was an 89-year-old woman presents to the emergency department after sustaining a fall. her vital signs are within appropriate limits for her age, the glasgow coma scale assessment is 7, and a ct scan reveals an intracranial hemorrhage. after obtaining labs, her estimated creatinine clearance (crcl) is calculated to be 35 ml/min. she does not require surgical intervention and the neurotrauma team decides to initiate levetiracetam for one week to prevent seizure. what is the maximum levetiracetam dose this patient may receive?